2013 AMC 8 Problems/Problem 15

Revision as of 15:34, 5 November 2017 by Destroyer2000 (talk | contribs) (Solution)

Problem

$6\cdot6=36$

Solve for s, x, and r.

See Also

2013 AMC 8 (ProblemsAnswer KeyResources)
Preceded by
Problem 14
Followed by
Problem 16
1 2 3 4 5 6 7 8 9 10 11 12 13 14 15 16 17 18 19 20 21 22 23 24 25
All AJHSME/AMC 8 Problems and Solutions

The problems on this page are copyrighted by the Mathematical Association of America's American Mathematics Competitions. AMC logo.png